Plan ahead for the next school year. Schedule your class today!

G
Topic
First Poster
Last Poster
IMO Shortlist 2010 - Problem G1
Amir Hossein   142
N 17 minutes ago by Fly_into_the_sky
Let $ABC$ be an acute triangle with $D, E, F$ the feet of the altitudes lying on $BC, CA, AB$ respectively. One of the intersection points of the line $EF$ and the circumcircle is $P.$ The lines $BP$ and $DF$ meet at point $Q.$ Prove that $AP = AQ.$

Proposed by Christopher Bradley, United Kingdom
142 replies
Amir Hossein
Jul 17, 2011
Fly_into_the_sky
17 minutes ago
A geometry problem
Lttgeometry   0
an hour ago
Let triangle $ABC$ have $(w_A)$ as the $A$-mixtilinear incircle, and let $A'$ be the tangency point of $(w_A)$ with the circumcircle $(O)$. Let $AA''$ be a diameter of $(w_A)$. Define $B''$, $C''$ similarly. Prove that the lines $AA''$, $BB''$, and $CC''$ are concurrent.
0 replies
Lttgeometry
an hour ago
0 replies
Hard functional equation
Sardor   20
N an hour ago by player-019
Source: IZHO2015.P3
Find all functions $ f\colon \mathbb{R} \to \mathbb{R} $ such that $ f(x^3+y^3+xy)=x^2f(x)+y^2f(y)+f(xy) $, for all $ x,y \in \mathbb{R} $.
20 replies
Sardor
Jan 13, 2015
player-019
an hour ago
P2 Cono Sur 2021
Leo890   10
N an hour ago by lendsarctix280
Source: Cono Sur 2021 P2
Let $ABC$ be a triangle and $I$ its incenter. The lines $BI$ and $CI$ intersect the circumcircle of $ABC$ again at $M$ and $N$, respectively. Let $C_1$ and $C_2$ be the circumferences of diameters $NI$ and $MI$, respectively. The circle $C_1$ intersects $AB$ at $P$ and $Q$, and the circle $C_2$ intersects $AC$ at $R$ and $S$. Show that $P$, $Q$, $R$ and $S$ are concyclic.
10 replies
Leo890
Nov 30, 2021
lendsarctix280
an hour ago
Prove that $\angle FAC = \angle EDB$
micliva   32
N an hour ago by Fly_into_the_sky
Source: All-Russian Olympiad 1996, Grade 10, First Day, Problem 1
Points $E$ and $F$ are given on side $BC$ of convex quadrilateral $ABCD$ (with $E$ closer than $F$ to $B$). It is known that $\angle BAE = \angle CDF$ and $\angle EAF = \angle FDE$. Prove that $\angle FAC = \angle EDB$.

M. Smurov
32 replies
micliva
Apr 18, 2013
Fly_into_the_sky
an hour ago
Peru IMO TST 2023
diegoca1   1
N 2 hours ago by MathLuis
Source: Peru IMO TST 2023 D1 P2
Let $n$ be a positive integer. On an $n \times n$ board, players $A$ and $B$ take turns in a game. On each turn, a player selects an edge of the board (not on the board border) and makes a cut along that edge. If after the move the board is split into more than one piece, then that player loses the game.

Player $A$ moves first. Depending on the value of $n$, determine whether one of the players has a winning strategy.
1 reply
diegoca1
Yesterday at 8:03 PM
MathLuis
2 hours ago
Purely projective statement
ChimkinGang   6
N 2 hours ago by axolotlx7
Source: Own
Let $ABCD$ be a quadrilateral with $S=AD\cap BC$, $T=AB\cap CD$, and $X=AC\cap BD$. Let $P$ be a point in the plane not on $TX$, $Q=BP\cap TX$, $R=SP\cap TX$, and $Q'$ be the point on $TX$ such that $(QQ';TX)=-1$. If $U=BD\cap PQ'$ and $V=AP\cap DR$, show that $U$, $V$, and $T$ are collinear.
6 replies
ChimkinGang
Jun 15, 2025
axolotlx7
2 hours ago
Rhombus EVAN
62861   72
N 3 hours ago by fearsum_fyz
Source: USA January TST for IMO 2017, Problem 2
Let $ABC$ be a triangle with altitude $\overline{AE}$. The $A$-excircle touches $\overline{BC}$ at $D$, and intersects the circumcircle at two points $F$ and $G$. Prove that one can select points $V$ and $N$ on lines $DG$ and $DF$ such that quadrilateral $EVAN$ is a rhombus.

Danielle Wang and Evan Chen
72 replies
62861
Feb 23, 2017
fearsum_fyz
3 hours ago
Probablity problem
AlanLG   1
N 3 hours ago by AlexCenteno2007
Source: Mathematics Regional Olympiad of Mexico Southeast 2019 P3
Eight teams are competing in a tournament all against all (every pair of team play exactly one time among them). There are not ties and both results of every game are equally probable. What is the probability that in the tournament every team had lose at least one game and won at least one game?
1 reply
AlanLG
Oct 23, 2021
AlexCenteno2007
3 hours ago
Four variables (5)
Nguyenhuyen_AG   1
N 3 hours ago by arqady
Let $a,\,b,\,c,\,d$ be non-negative real numbers, such that $a+b+c+d=4.$ Prove that
\[52  + 17(\sqrt a + \sqrt b + \sqrt c + \sqrt d)^2\geqslant 9(ab+ bc + ca + da  + db + dc)^2.\]hide
1 reply
Nguyenhuyen_AG
5 hours ago
arqady
3 hours ago
N-M where M,N two 5-digit ''consecutive'' palindromes
parmenides51   1
N 3 hours ago by AlexCenteno2007
Source: Mathematics Regional Olympiad of Mexico Center Zone 2018 P1
Let $M$ and $N$ be two positive five-digit palindrome integers, such that $M <N$ and there is no other palindrome number between them. Determine the possible values of $N-M$.
1 reply
parmenides51
Nov 13, 2021
AlexCenteno2007
3 hours ago
Jbmo 2011 Problem 4
Eukleidis   13
N May 5, 2025 by Adventure1000
Source: Jbmo 2011
Let $ABCD$ be a convex quadrilateral and points $E$ and $F$ on sides $AB,CD$ such that
\[\tfrac{AB}{AE}=\tfrac{CD}{DF}=n\]

If $S$ is the area of $AEFD$ show that ${S\leq\frac{AB\cdot CD+n(n-1)AD^2+n^2DA\cdot BC}{2n^2}}$
13 replies
Eukleidis
Jun 21, 2011
Adventure1000
May 5, 2025
Jbmo 2011 Problem 4
G H J
Source: Jbmo 2011
The post below has been deleted. Click to close.
This post has been deleted. Click here to see post.
Eukleidis
78 posts
#1 • 1 Y
Y by Adventure10
Let $ABCD$ be a convex quadrilateral and points $E$ and $F$ on sides $AB,CD$ such that
\[\tfrac{AB}{AE}=\tfrac{CD}{DF}=n\]

If $S$ is the area of $AEFD$ show that ${S\leq\frac{AB\cdot CD+n(n-1)AD^2+n^2DA\cdot BC}{2n^2}}$
This post has been edited 1 time. Last edited by Eukleidis, Jun 21, 2011, 5:08 PM
Z K Y
The post below has been deleted. Click to close.
This post has been deleted. Click here to see post.
SCP
1502 posts
#2 • 2 Y
Y by Adventure10, Mango247
Eukleidis wrote:
Let ABCD be a convex quadrilateral and points E and F on sides AB,CD such that
\[\tfrac{AB}{AE}=\tfrac{CD}{CF}=n\]

If S is the area of AEFD show that ${S\leq\frac{AB\cdot CD+n(n-1)AD^2+n^2DA\cdot BC}{2n^2}}$

Let it be rectangle with $AB=CD=100,AD=BC=1$ and $n=100$ than we have $50\le 0.5+\frac{2n^2-n}{2n^2}<1.5$ hence a mistake in question?
Z K Y
The post below has been deleted. Click to close.
This post has been deleted. Click here to see post.
Eukleidis
78 posts
#3 • 2 Y
Y by Adventure10, Mango247
I made a mistake. Its not CF but DF. Sorry.
Z K Y
The post below has been deleted. Click to close.
This post has been deleted. Click here to see post.
iron
3 posts
#4 • 2 Y
Y by Adventure10, Mango247
There should be ${nDA\cdot BC}$ instead of ${n^2DA\cdot BC}$ it seems.
We can get it using Ptolemy's inequality
${AF\cdot ED\leq AE\cdot DF+AD\cdot EF}$
and other noname inequality
$\sqrt{(a+b)^2 +(c+d)^2}\leq\sqrt{a^2 +c^2 }+\sqrt{b^2 +d^2 }.$
Z K Y
The post below has been deleted. Click to close.
This post has been deleted. Click here to see post.
mavropnevma
15142 posts
#5 • 3 Y
Y by horizon, jam10307, Adventure10
Indeed, it is just an exercise in known formulae (by the way, that "noname" inequality is Minkowski's, also known as the "triangle" inequality :) ). And indeed, the $n^2DA\cdot BC$ should be $nDA\cdot BC$; mind you, when it works for the latter, it will a fortiori work for the former!

The area $S$ of the quadrilateral $AEFD$ is $S = \dfrac {1} {2} AF\cdot DE\cdot \sin\angle(AF,DE) \leq \dfrac {1} {2} AF\cdot DE \leq \dfrac {1} {2} (AE\cdot DF + AD\cdot EF)$, by the known fact that the area of a convex quadrilateral is half the product of its diagonals with the sine of their angle, followed by the fact that the sine of an angle is at most $1$, followed by Ptolemy's inequality.

Now, $AE\cdot DF = \dfrac {AB\cdot DC} {n^2}$, so if we prove that $AD\cdot EF \leq \dfrac {(n-1)AD^2 + AD\cdot BC} {n}$, i.e. $EF \leq \dfrac {(n-1)AD + BC} {n}$, that will be enough. But this is a simple vectorial computation.

Denote by lowercase letters the position vectors of the uppercase points. Then $e = a+\dfrac {1} {n} (b-a)$ and $f = d+\dfrac {1} {n} (c-d)$, so $e-f = (a-d) + \dfrac {1} {n} ((b-c) - (a-d)) = \dfrac {n-1} {n} (a-d) + \dfrac {1} {n} (b-c)$.
Then $EF = |e-f| =$ $ \left |\dfrac {n-1} {n} (a-d) + \dfrac {1} {n} (b-c)\right | \leq$ $ \dfrac {n-1} {n} |a-d| + \dfrac {1} {n} |b-c| =$ $ \dfrac {n-1} {n} AD + \dfrac {1} {n} BC$, by the triangle inequality.
Z K Y
The post below has been deleted. Click to close.
This post has been deleted. Click here to see post.
iron
3 posts
#6 • 2 Y
Y by Adventure10, Mango247
Quote:
also known as the "triangle" inequality
I know but "triangle" isn't a name actually. Oh well, just kidding :roll:
Z K Y
The post below has been deleted. Click to close.
This post has been deleted. Click here to see post.
Merlinaeus
163 posts
#7 • 2 Y
Y by Adventure10, Mango247
iron wrote:
and other noname inequality
$\sqrt{(a+b)^2 +(c+d)^2}\leq\sqrt{a^2 +c^2 }+\sqrt{b^2 +d^2 }.$
mavropnevma wrote:
Indeed, it is just an exercise in known formulae (by the way, that "noname" inequality is Minkowski's, also known as the "triangle" inequality
It's not quite Minkowski in the standard form in which that is usually quoted, and why is it the triangle inequality?
It is, however, straight AM/GM if you just square each side a couple of times.
Merlin
Z K Y
The post below has been deleted. Click to close.
This post has been deleted. Click here to see post.
mavropnevma
15142 posts
#8 • 3 Y
Y by Adventure10, Mango247, and 1 other user
Consider points $O(0,0), A(a,c), B(-b,-d)$. Then $\sqrt{(a+b)^2 + (c+d)^2} = AB$, $\sqrt{a^2 + c^2} = AO$, $\sqrt{b^2 + d^2} = OB$, so the inequality is equivalent to $AB \leq AO + OB$, which, as far as I remember, is the triangle inequality, expressed in the Euclidean norm (in any dimension) by Minkovski's inequality. The fact it is not appearing in its usual form
$\sqrt{(x_1-x_3)^2 + (y_1-y_3)^2} \leq$ $ \sqrt{(x_1-x_2)^2 + (y_1-y_2)^2} + \sqrt{(x_2-x_3)^2 + (y_2-y_3)^2}$,
but is just a particular form, does not rob it of its origin.
Z K Y
The post below has been deleted. Click to close.
This post has been deleted. Click here to see post.
paul1703
222 posts
#9 • 3 Y
Y by coldheart361, Adventure10, Mango247
Marius Stanean's proof
Take points $X,Y$ such as $AX\parallel CD \wedge FX\parallel DA,\; BY\parallel CD \wedge FY\parallel BC\Rightarrow ADFX,BCFY$ are parallelograms.
We have
$\left.\begin{array}{l}
\frac{AX}{BY}=\frac{DF}{FC}=\frac{1}{n-1}=\frac{AE}{BE}\\
AX\parallel BY\end{array}\right\}\Rightarrow \triangle AXE\sim\triangle BYE\Rightarrow X,E,Y$ collinear $\frac{EX}{EY}=\frac{1}{n-1}\,.$

Fie $Z\in FE,\;(E\in(FZ))$ such as $\frac{EF}{EZ}=\frac{1}{n-1}\Rightarrow \triangle EFX\sim\triangle EZY\Rightarrow \frac{FX}{ZY}=\frac{1}{n-1}$.

In triangle $ZFY$
$FZ\le FY+ZY\Leftrightarrow \boxed{n\cdot EF\le BC+(n-1)AD}\;\;\;(*)$
$S=\frac{AF\cdot DE}{2}\cdot \sin\angle (AF,DE)\le \frac{AF\cdot DE}{2}\stackrel{\mbox{\tiny I.Ptolemeu}}{\le} \frac{AD\cdot EF+AE\cdot DF}{2}=$ $\frac{n^2\cdot AD\cdot EF+AB\cdot CD}{2n^2}\stackrel{(*)}{\le}\frac{AB\cdot CD+n(n-1)DA^{2}+n\cdot DA\cdot BC}{2n^2}\,.$
Z K Y
The post below has been deleted. Click to close.
This post has been deleted. Click here to see post.
mavropnevma
15142 posts
#10 • 1 Y
Y by Adventure10
So this proof offers a synthetic argument for $EF \leq \dfrac {(n-1)AD + BC} {n}$ (having as last recourse the triangle inequality - what else?); otherwise it is difficult to foresee a change in the chain of formulae.
Z K Y
The post below has been deleted. Click to close.
This post has been deleted. Click here to see post.
paul1703
222 posts
#11 • 2 Y
Y by Adventure10, Mango247
The solution is originaly from http://forum.gil.ro/viewtopic.php?f=19&t=1198 .I don't understand what part of the proof is incomplete?
Z K Y
The post below has been deleted. Click to close.
This post has been deleted. Click here to see post.
mavropnevma
15142 posts
#12 • 2 Y
Y by Adventure10, Mango247
No part is incomplete (never said that) - it's just that the chain of ideas must be the same.
Z K Y
The post below has been deleted. Click to close.
This post has been deleted. Click here to see post.
dr_Civot
354 posts
#13 • 5 Y
Y by KRIS17, Profserhat, Adventure10, Mango247, and 1 other user
Eukleidis wrote:
Let ABCD be a convex quadrilateral and points E and F on sides AB,CD such that
\[\tfrac{AB}{AE}=\tfrac{CD}{DF}=n\]

If S is the area of AEFD show that ${S\leq\frac{AB\cdot CD+n(n-1)AD^2+nDA\cdot BC}{2n^2}}$
PART 1
PART 2
Z K Y
The post below has been deleted. Click to close.
This post has been deleted. Click here to see post.
Adventure1000
6 posts
#14
Y by
is here $AD\parallel EF \parallel BC$
Z K Y
N Quick Reply
G
H
=
a